Which one of the following could be true?

zia305 on August 29, 2020

Explanation

Can someone please explain the right answer?

Reply
Create a free account to read and take part in forum discussions.

Already have an account? log in

Emil-Kunkin on December 25 at 09:02PM

We know automatically that F and H cannot serve on all three, as this would mean another cannot serve on any, so we only need consider CDE.

C is impossible since either m or p must be on all three, and G cannot be on all three.
E is similarly impossible, because if m and p are both in three slots, we don't have enough slots left.

D could happen
1: P m f
2: P m h
3: P I g